1answer.
Ask question
Login Signup
Ask question
All categories
  • English
  • Mathematics
  • Social Studies
  • Business
  • History
  • Health
  • Geography
  • Biology
  • Physics
  • Chemistry
  • Computers and Technology
  • Arts
  • World Languages
  • Spanish
  • French
  • German
  • Advanced Placement (AP)
  • SAT
  • Medicine
  • Law
  • Engineering
vovikov84 [41]
2 years ago
8

A loop rests in the plane of a page of textbook while a magnetic field is directed into the page. A clockwise current is induced

.
Physics
1 answer:
weqwewe [10]2 years ago
7 0

(b) when the magnetic field gets stronger.

correct option :

When a loop rest in the plane of the page and magnetic field is directed into the page then we will apply here right hand thumb rule in which thumb represent the direction of magnetic field and curling fingers represent direction of induced current. So, in this case the direction of magnetic field is into the page and curling fingers shows induced current in clockwise direction.

  Thus, option (b) is correct.

Incorrect options:

   option (A) is incorrect because when magnetic field will change  

     according to flux.

   option (C) is incorrect because field is stronger so it does not matter

     size of loop increases or decreases.

   option (D) is incorrect because it moves any other side the field will be

     stronger.

Learn more about magnetic field induction here:

 brainly.com/question/13610297

   #SPJ4

[Your question is incomplete, but most probably your full question was-Check all that apply:

A. when the magnetic field is tilted so it is no longer perpendicular to the page

B. when the magnetic field gets stronger

C. when the size of the loop decreases

D. when the loop is moved sideways across the page]

You might be interested in
Which of the following is not a way that astronomers can find how much dark matter there is in cluster of galaxies?
Tpy6a [65]

Answer:

<em>b. Observe the radio waves coming from all dark matter; from the strength of the radio waves from each cluster, estimate the amount of dark matter needed to produce them.</em>

<em></em>

Explanation:

The universe is thought to be made up of 85% dark matters. <em>Dark matter is called dark because it does not appear to interact with the electromagnetic field, which means it doesn't absorb, reflect or emit electromagnetic radiation, and is therefore difficult to detect. This means that option b is wrong since radio wave is an electromagnetic wave</em>. Dark matter is a form of matter that makes up  about a quarter of the total mass–energy density of the universe. Dark matter was theorized due a variety of astrophysical observations and gravitational effects that cannot be explained by accepted theories of gravity unless there were more matter in the universe than can be seen.

6 0
3 years ago
3000 N is exerted for 4.0 seconds on a 9500 kg object.<br><br> What is the change in momentum?
WARRIOR [948]
Force is the change in momentum over a specific time. The change of momentum is therefore the force multiplied by the time that the force acts, so 3000x4.0=12000 N s=12000 kg m/s
8 0
2 years ago
Which Table of ordered pairs represent proportional relationship
Wewaii [24]

Answer:

Ratio table of ordered pairs represent proportional relationship .

<em>Hope </em><em>it</em><em> is</em><em> helpful</em><em> to</em><em> you</em>

5 0
3 years ago
72.0-kg person pushes on a small doorknob with a force of 5.00 N perpendicular to the surface of the door. The doorknob is locat
expeople1 [14]

Answer:I=2 kg-m^2

Explanation:

Given

mass m=72 kg

Force F=5 N

door knob is located at a distance of r=0.8 m from axis

Angular acceleration of door \alpha =2 rad/s^2

Torque T=I\alpha =F\times r

where I=moment of inertia

5\times 0.8=I\times 2

I=2 kg-m^2

4 0
3 years ago
How did the continental Shelf form? Please help, thank you!! :)
shutvik [7]
The movements of the tectonic plates
6 0
3 years ago
Other questions:
  • If an object has a mass of 81 kg, what is its approximate weight on earth?
    9·1 answer
  • A horizontal power line carries a current of 7250 A from south to north. Earth's magnetic field (65.7 µT) is directed toward the
    13·1 answer
  • Andre the Giant and Thumbelina are standing in the middle of an ice-skating rink facing each other. Andre the Giant pushes Thumb
    6·2 answers
  • A kayaker needs to paddle north across a 85 m wide harbor. The tide is going out, creating a tidal current that flows to the eas
    7·1 answer
  • How do you know the force for an acceleration of zero?
    8·1 answer
  • Calculate the moment of inertia and the rotational kinetic energy for the following objects spinning about a central axis (in un
    7·1 answer
  • 4) A force of 500 N acts on an area of 0.05m2. Find the pressure in Pascal.
    13·1 answer
  • How did the iron filing patterns show attractive forces between magnetic poles?
    15·2 answers
  • Calculate the net force acting on the box in the following. refer to the screenshot below.
    12·1 answer
  • could you help with question 5. Your solutions to the word problems in volving Newton's Laws should have the following features:
    15·1 answer
Add answer
Login
Not registered? Fast signup
Signup
Login Signup
Ask question!